Math, asked by yashchoudhary97, 3 months ago

in fig given that the ABCD is a rhombus find: (1) AB (2) m angle ABC​

Attachments:

Answers

Answered by nasrinbrngps
0

Answer:

Step-by-step explanation:

jahfff5+2 =7. 5+4=?

Similar questions